Fix subscripting (r_12 -> r_{12}) in Serway and Jewett v8's 25.21.
[course.git] / latex / problems / Serway_and_Jewett_4_wking / problem12.06.T.tex
1 \begin{problem}
2 A thin, rigid rod $L = 8.4\U{m}$ long pivots freely about one end.
3 The rod is initially deflected $\theta_i = 6.4\dg$ from the vertical
4 with an angular velocity of $\dt\theta_i = 2.7\dg\text{/s}$.
5 \Part{a} Determine the time dependence $\theta(t)$.
6 \Part{b} By what angle is the rod deflected at $t=8.9\U{s}$?
7
8 Hint: you might want to review torque and moments of inertia in
9 Chapter 10.
10 \end{problem} % combines the second part of P12.6 with Examp 12.6 and Tbl 10.2.
11
12 \begin{solution}
13 \begin{center}
14 \begin{asy}
15 import Mechanics;
16 real u = 1cm;
17
18 real a=6.4; // degrees
19 real force=2u; // magnitude
20 Pendulum p = makePendulum(angleDeg=a, length=2u,
21     stringL=Label("$L$", align=E, embed=Shift));
22 p.mass.lc.radius = 0;
23 Vector fg = Force(p.mass.center()/2, dir=-90, mag=force,
24     L=Label("$F_g$", position=EndPoint, align=W));
25 Vector fgtan = Force(p.mass.center()/2, dir=a-180, mag=force*sin(a),
26     L=Label("$F_{\tan}$", position=EndPoint, align=W));
27 Vector v = Velocity(p.mass.center(), dir=a, mag=0.5u,
28     L=Label("$v$", position=EndPoint, align=S));
29
30 fg.draw();
31 fgtan.draw();
32 v.draw();
33 p.draw(drawVertical=true);
34 \end{asy}
35 \end{center}
36 The only force on the rod is from gravity, with $mg$ pulling the rods
37 center of mass downward.  Only the portion of this force that is
38 perpendicular to the rod itself (tangential to the circle the rod
39 sweeps out) affects its rotation.  The torque on the rod is thus
40 \begin{equation}
41   \tau = -F_{\tan} \cdot \frac{L}{2}
42     = -F_g \sin(\theta) \cdot \frac{L}{2}
43     = -\frac{mgL}{2}\sin(\theta)
44     \approx -\frac{mgL}{2}\theta\;,
45 \end{equation}
46 where we used the small angle approximation $\sin(\theta) \approx
47 \theta$ for the last step.
48
49 The equation of motion is then
50 \begin{equation}
51   \tau = I\ddt\theta = \frac{1}{3}mL^2\ddt\theta\;,
52 \end{equation}
53 because the moment of inertia of a rod rotating about it's end is $I =
54 \frac{1}{3}mL^2$ (Table 10.2).
55
56 Combining the two expressions of $\tau$ we have
57 \begin{align}
58   -\frac{mgL}{2}\theta &= \frac{1}{3}mL^2\ddt\theta \\
59   -\frac{3g}{2L}\theta &= \ddt\theta\;.
60 \end{align}
61 Comparing this formula to Equation 12.5 for a general simple harmonic oscillator
62 \begin{equation}
63   \ddt x = -\omega^2 x\;,
64 \end{equation}
65 we see by matching that
66 \begin{equation}
67   \omega = \sqrt{\frac{3g}{2L}} \approx \ans{1.323\U{rad/s}}\;.
68 \end{equation}
69 We can plug this $\omega$ into Equation 12.6
70 \begin{equation}
71   \theta(t) = A \cos(\omega t + \phi)\;,
72 \end{equation}
73 where $A$ and $\psi$ are determined by the initial conditions (see Example 12.3)
74 \begin{align}
75   \theta_i &= 6.4\dg\text{/s} \cdot \frac{\pi}{180\dg} \approx 111.7\U{mrad} \\
76   \dt\theta_i &= 2.7\dg\text{/s} \cdot \frac{\pi}{180\dg} \approx 47.12\U{mrad/s} \\
77   \phi &= \arctan\p({\frac{-\dt\theta_i}{\omega\theta_i}}) \approx -308.7\U{mrad} \approx -17.69\dg \\
78   A &= \sqrt{\theta_i^2 + \p({\frac{\dt\theta_i}{\omega}})^2} \approx 117.2\U{mrad} \approx 6.718\dg\\
79   \theta(t) &\approx \ans{0.1172\cos\p({1.323t-0.3087})}\;.
80 \end{align}
81
82 \Part{b}
83 Plugging in $t=8.9\U{s}$ yields
84 \begin{equation}
85   \theta(t=8.9\U{s}) = 0.1172\cos\p({1.323 \cdot 8.9 - 0.3087})
86     \approx \ans{53\U{mrad}}
87     \approx \ans{3.0\dg}\;.
88 \end{equation}
89 \end{solution}